Find the center of mass of a plate

In summary: Right.In summary, the center of mass of the next plate would be higher if the density per unit mass is greater.
  • #1
Davidllerenav
424
14

Homework Statement


Find the center of mass of the next plate if:
A) Is homogeneous
B) Its density per unit mass is ##\sigma=Axy##, where ##A## is a constant.
Capture.PNG


Homework Equations


##X_{cm}=\frac{\int\sigma x dA}{\int\sigma dA}##
##Y_{cm}=\frac{\int\sigma y dA}{\int\sigma dA}##

The Attempt at a Solution


I did part A) as is shown in the picture below:
Image.jpg

Image (2).jpg

Am I correct?
My main problem is with part B), I tried to do it as I did part A), I found ##X_{cm} without any problems, like this:
Image (2)).jpg

But when I tried to find ##Y_{cm}## I got stucked with this:

##Y_{cm}=\frac{\int_{0}^7 \sigma ydA_1+ \int_{1}^7 \sigma ydA_2}{\int_{0}^7 \sigma dA_1 + \int_{1}^7 \sigma dA_2}=\frac{A\left[ \int_{0}^7 y^2\sqrt{y-1}dy + \int_{1}^7 y^2(y-1)dy\right]}{A\left[ \int_{0}^7 y\sqrt{y-1}dy + \int_{1}^7 y(y-1)dy\right]}##, but when I tried to integrate ##\int_{0}^7 y^2\sqrt{y-1}dy## and ##\int_{0}^7 y\sqrt{y-1}dy##, I ended up with imaginary numbers, what am I doing wrong? Hope you can help me.
 

Attachments

  • Capture.PNG
    Capture.PNG
    2.5 KB · Views: 859
  • Image.jpg
    Image.jpg
    86 KB · Views: 575
  • Image (2).jpg
    Image (2).jpg
    92.8 KB · Views: 555
  • Image (2)).jpg
    Image (2)).jpg
    109.7 KB · Views: 567
Physics news on Phys.org
  • #2
Think more carefully about what your ∫(x2+1).dx and ∫(x2+1)x.dx are actually measuring.
 
  • #3
haruspex said:
Think more carefully about what your ∫(x2+1).dx and ∫(x2+1)x.dx are actually measuring.
##\int (x^2+1)dx## is measuring the total area of the plate, while ##\int (x^2+1)xdx## is measuring the area of a small piece of the total area.
 
  • #4
Davidllerenav said:
##\int (x^2+1)dx## is measuring the total area of the plate,
No.
Look at your diagram for the x>0 section. What is the length in the y direction of the strip width dx?
 
  • #5
haruspex said:
No.
Look at your diagram for the x>0 section. What is the length in the y direction of the strip width dx?
I think it would be ##7-(x^2+1)##, right?
 
  • #6
Davidllerenav said:
I think it would be ##7-(x^2+1)##, right?
Right, so what should your integrals be?
 
  • #7
haruspex said:
Right, so what should your integrals be?
Oh, I see know, I was calculating the area underneath the curve, so I need to write ##7-x^2-1## instead to calculate the are between the line ##y=7## and the curve.
I think the integrals would be like this, am I correct?
Image (4).jpg


How would it be with ##Y_{cM}##?
 

Attachments

  • Image (4).jpg
    Image (4).jpg
    79.6 KB · Views: 444
  • #8
Davidllerenav said:
the integrals would be like this, am I correct?
Yes, that answer looks better.
Davidllerenav said:
How would it be with ##Y_{cM}##?
Please post a revised attempt. As a check, should it be more or less than 3.5?
 
  • #9
haruspex said:
Please post a revised attempt.
I'm trying to solve it. The length in the x direction of the strip width dy would be ##x##, right? And ##x=\sqrt{y-1}##.
haruspex said:
As a check, should it be more or less than 3.5?
I think that it should be less than 3.5, but not too much.
 
Last edited:
  • #10
Davidllerenav said:
I think that it should be less than 3.5, but not too much.
Consider the two parts separately. How would the two y coordinates of the mass centres compare with 3.5 individually?
 
  • #11
haruspex said:
Consider the two parts separately. How would the two y coordinates of the mass centres compare with 3.5 individually?
Well, the coordinates of the mass center of the rectangle would be ##(2.5,3.5)## and for the parabola, I guess the would be less.
 
  • #12
Davidllerenav said:
The length in the x direction of the strip width dy would be ##x##, right? And ##x=\sqrt{y-1}##.
@haruspex Am I correct with this?
 
  • #13
Davidllerenav said:
I'm trying to solve it. The length in the x direction of the strip width dy would be ##x##, right? And ##x=\sqrt{y-1}##.
Yes, but only for y in a certain range.
Davidllerenav said:
for the parabola, I guess the would be less.
The y coordinate would be less? Doesn't look that way to me.
 
  • #14
haruspex said:
Yes, but only for y in a certain range.
Yes, y would be between 1 and 7. So is my calculation of ##Y_{cm}## correct?
haruspex said:
The y coordinate would be less? Doesn't look that way to me.
Well, then I think I didn't understand the question.
 
  • #15
Davidllerenav said:
Well, then I think I didn't understand the question.
Consider the complete rectangle, 2x7, with lower left corner at the origin. The y coordinate of its mass centre would be 3.5, yes? To get the actual shape you have to trim off the piece below the parabola. Wouid that raise or lower the mass centre?
 
  • #16
haruspex said:
Consider the complete rectangle, 2x7, with lower left corner at the origin. The y coordinate of its mass centre would be 3.5, yes? To get the actual shape you have to trim off the piece below the parabola. Wouid that raise or lower the mass centre?
That would raise the center of mass, so the y component would be greater than 3.5.
 
  • #17
Davidllerenav said:
That would raise the center of mass, so the y component would be greater than 3.5.
Right.
 
  • #18
haruspex said:
Right.
Ok, so am I correct with how I calculated ##Y_{cm}## initially?
Also, how do I calculate the center of mass when the density isn't constant, i.e. part B)? I tried to solve it as I did with part A, but ##Y_{cm}## gives me imaginary numbers.
 
Last edited:
  • #19
Davidllerenav said:
so am I correct with how I calculated ##Yc_{cm} ##initially?
Since you got a value <3.5, no. The method looks ok so there must be an arithmetical error.
By the way, for both x and y coordinate of CoM, the denominator is the area of the plate, so you don’t need to calculate that twice.
 
Last edited:
  • #20
haruspex said:
Since you got a value <3.5, no.
I think I know the problem. Do you see that I got a different value in the total mass integral? The problem is in the parabola, since I got 35 for the rectangle for both ##X_{cm}## and ##Y_{cm}##. The problem is that the equatio of the parabola ##y=x^2+1## isn't complete with ##x\in[0,2]##. So when I integrate ##\int_{0}^2 6-x^2 dx## I'm not getting the whole area, while when I integrate ##\int_{1}^7 \sqrt{y-1}dy## I'm indeed getting the whole area. This can be seen here:
bfcd8436-46ac-4eaf-9ed5-a862dab945b1.jpg

2073807a-8b11-494b-be4e-e068a795857e.jpg

How can I fix this? The only way I can think of would be to change the upper limit on ##X_cm## with 2.449 instead of 2.
 

Attachments

  • bfcd8436-46ac-4eaf-9ed5-a862dab945b1.jpg
    bfcd8436-46ac-4eaf-9ed5-a862dab945b1.jpg
    12 KB · Views: 435
  • 2073807a-8b11-494b-be4e-e068a795857e.jpg
    2073807a-8b11-494b-be4e-e068a795857e.jpg
    11.4 KB · Views: 394
  • #21
Davidllerenav said:
I think I know the problem. Do you see that I got a different value in the total mass integral? The problem is in the parabola, since I got 35 for the rectangle for both ##X_{cm}## and ##Y_{cm}##. The problem is that the equatio of the parabola ##y=x^2+1## isn't complete with ##x\in[0,2]##.
Ha! The problem is that the given diagram is wrong! At x=2, x2+1 is only 5, not 7.
My guess is that the distances 5 and 7 are swapped over. The plate should be 5cm high and 7+2cm wide. That fits better with the way it is drawn.
 
  • #22
haruspex said:
Ha! The problem is that the given diagram is wrong! At x=2, x2+1 is only 5, not 7.
My guess is that the distances 5 and 7 are swapped over. The plate should be 5cm high and 7+2cm wide. That fits better with the way it is drawn.
Yes you're right. That would be a big change though, I'll try to do it. Also, how should I solve the part where the plate isn't himoghomoge?
 
  • #23
Davidllerenav said:
Yes you're right. That would be a big change though, I'll try to do it. Also, how should I solve the part where the plate isn't himoghomoge?
It's just the same but replacing constant σ with Axy. Post what you get.
 
  • #24
haruspex said:
It's just the same but replacing constant σ with Axy. Post what you get.
I'll post as soon as I finish calculating with the correct measurements.
 
  • #25
@haruspex Ok, this is what I got:
Part A):
Image (5).jpg

For part B) I had the same problem, I ended up with a integral ##7\int_{0}^5 \sqrt{y-1}dy## and ##7\int_{0}^5 \sqrt{y-1}ydy## and on both I will get imaginary numbers, what am I doing wrong?
Image (6).jpg
 

Attachments

  • Image (5).jpg
    Image (5).jpg
    58.8 KB · Views: 363
  • Image (6).jpg
    Image (6).jpg
    82.1 KB · Views: 360
  • #26
Davidllerenav said:
what am I doing wrong?
The bound x2<y-1 applies to what range of y?
 
  • #27
haruspex said:
The bound x2<y-1 applies to what range of y?
It only works for ##y\geq 1##, that's why I'm getting imaginary numbers.
 
  • #28
Davidllerenav said:
It only works for ##y\geq 1##, that's why I'm getting imaginary numbers.
That's right, the constraint only applies for y>1, and that part of the plate only exists for y>1. So use the correct bounds on the integral.
 
  • #29
haruspex said:
That's right, the constraint only applies for y>1, and that part of the plate only exists for y>1. So use the correct bounds on the integral.
But isn't that integral supposed to go from 0 to 5? Since it is integrating the rectangle.
 
  • #30
Davidllerenav said:
But isn't that integral supposed to go from 0 to 5? Since it is integrating the rectangle.
You can run the integral from 0 if you wish, but the constraint 0<x2<y-1 only applies for y>1. So you need to break the integral into separate ranges 0 to 1 and 1 to 5. The 0 to 1 range is trivially null.

Wait - you said integrating the rectangle. You mean the x<0 part? That's not the bit that gives imaginary numbers.
 
  • #31
haruspex said:
You can run the integral from 0 if you wish, but the constraint 0<x2<y-1 only applies for y>1. So you need to break the integral into separate ranges 0 to 1 and 1 to 5. The 0 to 1 range is trivially null.
So I just need to integrate from 1 to 5 on both integrals?
 
  • #32
Davidllerenav said:
So I just need to integrate from 1 to 5 on both integrals?
The numerator and denominator integrals for x>0, yes.
 
  • #33
haruspex said:
The numerator and denominator integrals for x>0, yes.
Ok, so just to check, the two integrals on the left must be from 1 to 5? So I will have four integrals from 1 to 5?
Capture2.PNG
 

Attachments

  • Capture2.PNG
    Capture2.PNG
    11.8 KB · Views: 562
  • #34
Davidllerenav said:
Ok, so just to check, the two integrals on the left must be from 1 to 5? So I will have four integrals from 1 to 5?
View attachment 240464
No, some confusion here.
You have integrals that relate to the 7x5 rectangle on the left (x<0) and integrals that relate to the curved flange on the right (x from 0 to 2).
Those for the right involve the constraint x2>y-1 and therefore only apply for y=1 to 5.
Those for the left apply for y=0 to 5, but since they do not involve the constraint x2<y-1 they should not contain any square roots so should not give imaginary nunbers.
 
  • #35
haruspex said:
No, some confusion here.
You have integrals that relate to the 7x5 rectangle on the left (x<0) and integrals that relate to the curved flange on the right (x from 0 to 2).
Those for the right involve the constraint x2>y-1 and therefore only apply for y=1 to 5.
Those for the left apply for y=0 to 5, but since they do not involve the constraint x2<y-1 they should not contain any square roots so should not give imaginary nunbers.
But as you can see in here, I ended up with square roots on the left:
Capture3.PNG
 

Attachments

  • Capture3.PNG
    Capture3.PNG
    77.2 KB · Views: 554

Similar threads

  • Introductory Physics Homework Help
Replies
9
Views
1K
  • Introductory Physics Homework Help
Replies
1
Views
195
  • Introductory Physics Homework Help
Replies
6
Views
2K
  • Introductory Physics Homework Help
Replies
6
Views
486
  • Introductory Physics Homework Help
Replies
6
Views
925
  • Introductory Physics Homework Help
Replies
15
Views
2K
  • Introductory Physics Homework Help
Replies
3
Views
1K
  • Introductory Physics Homework Help
Replies
3
Views
242
  • Introductory Physics Homework Help
Replies
4
Views
1K
  • Introductory Physics Homework Help
Replies
19
Views
2K
Back
Top